Wednesday, March 15, 2017

No, we probably don’t live in a computer simulation

According to Nick Bostrom of the Future of Humanity Institute, it is likely that we live in a computer simulation. And one of our biggest existential risks is that the superintelligence running our simulation shuts it down.

The simulation hypothesis, as it’s called, enjoys a certain popularity among people who like to think of themselves as intellectual, believing it speaks for their mental flexibility. Unfortunately it primarily speaks for their lacking knowledge of physics.

Among physicists, the simulation hypothesis is not popular and that’s for a good reason – we know that it is difficult to find consistent explanations for our observations. After all, finding consistent explanations is what we get paid to do.

Proclaiming that “the programmer did it” doesn’t only not explain anything - it teleports us back to the age of mythology. The simulation hypothesis annoys me because it intrudes on the terrain of physicists. It’s a bold claim about the laws of nature that however doesn’t pay any attention to what we know about the laws of nature.

First, to get it out of the way, there’s a trivial way in which the simulation hypothesis is correct: You could just interpret the presently accepted theories to mean that our universe computes the laws of nature. Then it’s tautologically true that we live in a computer simulation. It’s also a meaningless statement.

A stricter way to speak of the computational universe is to make more precise what is meant by ‘computing.’ You could say, for example, that the universe is made of bits and an algorithm encodes an ordered time-series which is executed on these bits. Good - but already we’re deep in the realm of physics.

If you try to build the universe from classical bits, you won’t get quantum effects, so forget about this – it doesn’t work. This might be somebody’s universe, maybe, but not ours. You either have to overthrow quantum mechanics (good luck), or you have to use qubits. [Note added for clarity: You might be able to get quantum mechanics from a classical, nonlocal approach, but nobody knows how to get quantum field theory from that.]

Even from qubits, however, nobody’s been able to recover the presently accepted fundamental theories – general relativity and the standard model of particle physics. The best attempt to date is that by Xiao-Gang Wen and collaborators, but they are still far away from getting back general relativity. It’s not easy.

Indeed, there are good reasons to believe it’s not possible. The idea that our universe is discretized clashes with observations because it runs into conflict with special relativity. The effects of violating the symmetries of special relativity aren’t necessarily small and have been looked for – and nothing’s been found.

For the purpose of this present post, the details don’t actually matter all that much. What’s more important is that these difficulties of getting the physics right are rarely even mentioned when it comes to the simulation hypothesis. Instead there’s some fog about how the programmer could prevent simulated brains from ever noticing contradictions, for example contradictions between discretization and special relativity.

But how does the programmer notice a simulated mind is about to notice contradictions and how does he or she manage to quickly fix the problem? If the programmer could predict in advance what the brain will investigate next, it would be pointless to run the simulation to begin with. So how does he or she know what are the consistent data to feed the artificial brain with when it decides to probe a specific hypothesis? Where does the data come from? The programmer could presumably get consistent data from their own environment, but then the brain wouldn’t live in a simulation.

It’s not that I believe it’s impossible to simulate a conscious mind with human-built ‘artificial’ networks – I don’t see why this should not be possible. I think, however, it is much harder than many future-optimists would like us to believe. Whatever the artificial brains will be made of, they won’t be any easier to copy and reproduce than human brains. They’ll be one-of-a-kind. They’ll be individuals.

It therefore seems implausible to me that we will soon be outnumbered by artificial intelligences with cognitive skills exceeding ours. More likely, we will see a future in which rich nations can afford raising one or two artificial consciousnesses and then consult them on questions of importance.

So, yes, I think artificial consciousness is on the horizon. I also think it’s possible to convince a mind with cognitive abilities comparable to that of humans that their environment is not what they believe it is. Easy enough to put the artificial brain in a metaphoric vat: If you don’t give it any input, it would never be any wiser. But that’s not the environment I experience and, if you read this, it’s not the environment you experience either. We have a lot of observations. And it’s not easy to consistently compute all the data we have.

Besides, if the reason you build an artificial intelligences is consultation, making them believe reality is not what it seems is about the last thing you’d want.

Hence, the first major problem with the simulation hypothesis is to consistently create all the data which we observe by any means other than the standard model and general relativity – because these are, for all we know, not compatible with the universe-as-a-computer.

Maybe you want to argue it is only you alone who is being simulated, and I am merely another part of the simulation. I’m quite sympathetic to this reincarnation of solipsism, for sometimes my best attempt of explaining the world is that it’s all an artifact of my subconscious nightmares. But the one-brain-only idea doesn’t work if you want to claim that it is likely we live in a computer simulation.

To claim it is likely we are simulated, the number of simulated conscious minds must vastly outnumber those of non-simulated minds. This means the programmer will have to create a lot of brains. Now, they could separately simulate all these brains and try to fake an environment with other brains for each, but that would be nonsensical. The computationally more efficient way to convince one brain that the other brains are “real” is to combine them in one simulation.

Then, however, you get simulated societies that, like ours, will set out to understand the laws that govern their environment to better use it. They will, in other words, do science. And now the programmer has a problem, because it must keep close track of exactly what all these artificial brains are trying to probe.

The programmer could of course just simulate the whole universe (or multiverse?) but that again doesn’t work for the simulation argument. Problem is, in this case it would have to be possible to encode a whole universe in part of another universe, and parts of the simulation would attempt to run their own simulation, and so on. This has the effect of attempting to reproduce the laws on shorter and shorter distance scales. That, too, isn’t compatible with what we know about the laws of nature. Sorry.

Stephen Wolfram (from Wolfram research) recently told John Horgan that:
    “[Maybe] down at the Planck scale we’d find a whole civilization that’s setting things up so our universe works the way it does.”

I cried a few tears over this.

The idea that the universe is self-similar and repeats on small scales – so that elementary particles are built of universes which again contain atoms and so on – seems to hold a great appeal for many. It’s another one of these nice ideas that work badly. Nobody’s ever been able to write down a consistent theory that achieves this – consistent both internally and with our observations. The best attempt I know of are limit cycles in theory space but to my knowledge that too doesn’t really work.

Again, however, the details don’t matter all that much – just take my word for it: It’s not easy to find a consistent theory for universes within atoms. What matters is the stunning display of ignorance – for not to mention arrogance –, demonstrated by the belief that for physics at the Planck scale anything goes. Hey, maybe there’s civilizations down there. Let’s make a TED talk about it next. For someone who, like me, actually works on Planck scale physics, this is pretty painful.

To be fair, in the interview, Wolfram also explains that he doesn’t believe in the simulation hypothesis, in the sense that there’s no programmer and no superior intelligence laughing at our attempts to pin down evidence for their existence. I get the impression he just likes the idea that the universe is a computer. (Note added: As a commenter points out, he likes the idea that the universe can be described as a computer.)

In summary, it isn’t easy to develop theories that explain the universe as we see it. Our presently best theories are the standard model and general relativity, and whatever other explanation you have for our observations must first be able to reproduce these theories’ achievements. “The programmer did it” isn’t science. It’s not even pseudoscience. It’s just words.

All this talk about how we might be living in a computer simulation pisses me off not because I’m afraid people will actually believe it. No, I think most people are much smarter than many self-declared intellectuals like to admit. Most readers will instead correctly conclude that today’s intelligencia is full of shit. And I can’t even blame them for it.

130 comments:

  1. bee:

    just a correction. Stephen Wolfram does NOT believe that the universe IS a computer (or a cellular automaton for that matter). if you read the beginning section of chapter 7 in his quitec literally weighty book "A New Kind of Science", you'll find that he takes the role of the theoretical physicist to be (in agreement with the view of von Neumann) constructing models that predict experimentally observed phenomena (so he is, like you a phenomenologist if i understand what you mean when you say you are a phenomenologist). he says in his lectures, blogs and book that "the universe (or any specific phenomena) behaves AS IF it is.... he does not claim that nature does in fact, follow the behavior of models which are simplified representations of aspects of reality (e.g. stephen might say that just because you can describe the flight path of a frisbee using differential equations does not mean that a person or a dog actually solves differential equations when playing frisbee catch).

    best regards,

    richard

    ReplyDelete
  2. The computer simulation hypothesis is what you get when you cross philosophy with journalism.

    ReplyDelete
  3. Dear Sabine,

    I think the simulation hypothesis belongs with other skeptical scenarios: yes, it is logically possible that the world could be very different from how it appears to us now or how we model it so far, but, no, there is no good reason to focus on this particular possibility. We should assign some probability to it, and then get on with science and critical thinking based on the higher probability cases. Eric Schwitzgebel has recently defended such a view under the name "1% skepticism". If we ever get evidence corroborating one of the - what are now considered to be - skeptical scenarios, then we should update our probabilities and develop theories for it, but prior to that it makes no sense: without evidence, there is nothing to work with.
    One complication with this particular skeptical scenario is that Bostrom has offered an argument to the conclusion that we should assign high probability to the simulation hypothesis. Of course, there may be flaws in the argument, but even if you were to accept the argument, the fact that there is no direct evidence available seems a good reason for scientists not to spend too much time on it.
    So, I definitely understand why all the attention to the simulation hypothesis annoys you, and the above points would be my way of responding to someone who brings it up.

    Best wishes,
    Sylvia

    ReplyDelete
  4. An alternative way to refute the simulation hypothesis would be to estimate the computational requirements to run a simulation for the visible universe, even assuming qubits. What would be the memory requirement and can't we estimate the information content of the simulation vis a vis the simulated in terms of entropy. Perhaps that would show the meaninglessness of the simulation argument. It reminds me of Jorge Luis Borges short story on the country of cartographers who built a map as big as the country.

    ReplyDelete
  5. Future of Humanity is a mumble factory trading apocalypse for self-importance. It is vacuum eager for gilded containers, the macroeconomics of political connivance. Future of Humanity demands credulity for shaking an aspergillum dispensing ubiquitous dread.

    "today’s intelligencia is full of shit" and so willing to share, at gunpoint. Quis custodiet ipsos custodes? Entropy. Fear soft landings - the Third World.

    ReplyDelete
  6. Hi Richard,

    ok, thanks for pointing out. I should have been more careful phrasing that, will fix it.

    ReplyDelete
  7. Able,

    Well, Bostrom claims he's estimated it.

    ReplyDelete
  8. AT,

    Well, quite possibly you're not convinced there's no simulation because that wasn't what I was trying to argue. I was merely saying that the argument that it is likely we live in a simulation is wrong, or rather not even wrong - it's simply not an argument that lives up to the scientific standard. Maybe we live in a simulation - but I think it's unlikely for the reasons mentioned above: The difficulties in doing this seem to me vastly underestimated.

    Yes, I've meant to write a rant on Boltzmann brains for some while, thanks for reminding me of this...

    Best,

    B.

    ReplyDelete
  9. Very interesting, when it comes to deciphering junk from real science, you are my spirit animal, Sabine.

    ReplyDelete
  10. " It reminds me of Jorge Luis Borges short story on the country of cartographers who built a map as big as the country."

    As far as I know, this scenario first appeared in Sylvie and Bruno Concluded by Lewis Carroll. Interestingly, this book also contains the idea of a train running in a tunnel which is a chord of the Earth, powered by gravity, with the journey always taking the same amount of time (about 42 minutes), regardless of the length of the chord.

    ReplyDelete
  11. "Yes, I've meant to write a rant on Boltzmann brains for some while, thanks for reminding me of this... "

    Don't wait too long, otherwise one will materialize out of the void and write the post before you do!

    ReplyDelete
  12. Hello Bee,
    thank You for this Deep Thought(s) :=)
    Georg

    ReplyDelete
  13. Back when Bostrom first proposed this, I wrote a rebuttal, "Simulations and Reality in WYSIWYG Universes", exposing flaws in his math. (The paper was rejected by the journal he'd published his piece in, where he was, it appeared from the comments, the peer reviewer who rejected it. It was ultimately published in the SFWA Bulletin, in 2009, and is now available on Amazon/etc. as a little standalone thought piece ebook.)

    Bottom line, by my analysis, the odds are much higher that the universe is exactly as we see it, not a simulation. (Though some other interesting conclusions follow from the math, about the nature of such universes, and the end game for intelligent inhabitants.) :) It was a fun analysis.

    ReplyDelete
  14. @Able
    "An alternative way to refute the simulation hypothesis would be to estimate the computational requirements to run a simulation for the visible universe, even assuming qubits."

    That has actually been done by Seth Lloyd
    Computational capacity of the universe
    https://arxiv.org/abs/quant-ph/0110141

    I do not think he used qubits, but I am not sure. It is relatively easy to argue that, given the laws of nature, a simulation of the universe takes (much) more space than the universe itself. If you assume that the simulation runs according to other laws, anything goes.

    ReplyDelete
  15. Bostrom's simulation argument mistakenly uses probability theory to make statements about reality instead of statements about belief. If you say that the probability that we are living in a simulation is p, it means that you tested n universes and found pn of them to be simulations. In reality, no such test exists so it is not even a meaningful question.

    ReplyDelete
  16. " there’s a trivial way in which the simulation hypothesis is correct: You could just interpret the presently accepted theories to mean that our universe computes the laws of nature. Then it’s tautologically true that we live in a computer simulation."

    I wouldn't even go that far. Here's an argument against even this "trivial" point: https://arxiv.org/abs/1211.7081 . Whether or not you buy the argument, I hope you would at least concede that the statement "the universe computes itself like a computer" is not a tautology.

    ReplyDelete
  17. This comment has been removed by the author.

    ReplyDelete
  18. Just because the universe is computable (Lloyd says 10^120 quantum operations) does not mean it is computable in a way that is useful to us. Wolpert proves that two computers cannot mutually simulate each other, which implies that a computer cannot simulate itself. A computer cannot have enough memory to know its own state because it needs at least one more bit to make any observation from its simulation and then has to include that bit in its state. Any computer that models the exact physics of our universe would have to exist outside our observable universe.

    ReplyDelete
  19. Suggesting the universe we're in can't be a simulation by appealing to computational complexity limits based on what we see in this universe is a bit of a circular argument. 'It seems very difficult to me' is like a computer game character having a hard time believing in computer game designers. Unknown unknowns.

    I think the point of the hypothesis is that if it's possible to simulate experience (likely), and it's possible to nest simulations (also likely), it's a strange assumption to say you must be at the top of the tree. Nothing more controversial than that. Neither do I see any genuine showstopper in quantum mechanics. Maybe whatever box we are all running on really does simulate every single interaction! Or maybe it takes clever shortcuts, or fools us every time, or collapses wavefunctions on its own or whatever - we don't know anything about it. I personally find any of those ideas unlikely to the point of lunacy, but I don't have any ideological issue with it like you seem to, Sabine.

    ReplyDelete
  20. "It’s not that I believe it’s impossible to simulate a conscious mind with human-built ‘artificial’ networks – I don’t see why this should not be possible"

    I think Roger Penrose made pretty compelling arguments that it might very well be impossible. Everyone gets hung up on Gödel's Theorem here and seems to have missed the simpler arguments he made, which I thought were more powerful in some ways. To show that consciousness is not (fully) algorithmic in nature, we just need simple examples where we can show consciousness can do something an algorithm can't. Penrose lists some math problems that do not have algorithmic solutions (even in principle). Not only have we solved these problems, for some we've even proved they cannot be solved by an algorithm. The ones he talked about that come to mind are the halting problem, tiling problem and general Diophantine equations, although there were others, I believe.

    Sounds really hard to simulate a part of the universe that is non-algorithmic doing something non-algorithmic like proving a problem is non-algorithmic, by using an ... algorithm.

    Although, I seem to recall Penrose leaving open room for non-computational "algorithms", while admitting he has no idea what this would be like!

    ReplyDelete
  21. This comment has been removed by the author.

    ReplyDelete
  22. Matt Mahoney, this was an interesting statement:

    "A computer cannot have enough memory to know its own state because it needs at least one more bit to make any observation from its simulation and then has to include that bit in its state."

    Sounds like an possible argument against a computer becoming self-aware, at least fully?

    Then again, I'm not sure most humans are even close to fully self-aware!

    Anyhow, thanks for sharing, will have to look into this.

    ReplyDelete
  23. Something more about Borges. “Nosotros (la indivisa divinidad que opera en nosotros) hemos soñado el mundo. Lo hemos soñado resistente, misterioso, visible, ubicuo con el espacio y firme en el tiempo; pero hemos consentido en su arquitectura tenues y eternos intersticios de sinrazón para saber que es falso." J.L.Borges, 'Avatares de la Tortuga'

    "We (the undivided divinity that operates in us) have dreamed the world. We have dreamed it gnarly, mysterious, observable, continuous in space and reliable in time; but we have allowed into its architecture anomalous gaps of irrationality both indeterminate and timeless so we might know that it is a contrivance."

    ReplyDelete
  24. Hi Bee, what's your opinion of https://arxiv.org/abs/1210.1847 ?

    ReplyDelete
  25. It seems to me that there's another solution for the "accuracy of simulation" problem nobody talks about, a solution with amusing consequences.

    The beings-that-are-not-called-gods simulate each mind individually, and turn people off once they study too much physics and strain the simulation resources. The simulation of society includes physicists of course, but the simulated minds only enjoy popular articles and the fruits of technology. (The simulators might need to exclude psychologists and sociologists, and some of the more observant philosophers. But still, the simulated beings would far outnumber the physical ones.)

    Obviously, there's a very different distribution of professions on simulated beings than in our society, since the simulated physicists are killed early. But since the society is simulated, there is no need for it to match the distribution of brains.

    The degree of belief in this simulation hypothesis would thus depend on your occupation. If you are a physicist, the argument above would imply you must be a real live mind, living its life in the physical universe. But if you are only a reader of physics blogs, you may as well be simulated - simulating just your mind likely doesn't even require any quantum effect. This effect is stronger the less you interact with society, since you require less simulation resources this way (as society is made up, just for you). So in this scenario we can except the less socially-fluent non-physicists to believe the most that they are living in a simulation.

    ReplyDelete
  26. Sabine said…

    “The idea that our universe is discretized clashes with observations because it runs into conflict with special relativity. …the programmer could prevent simulated brains from ever noticing contradictions, for example contradictions between discretization and special relativity.”

    I wanted to ask that you consider discussing the issue above (discretization and special relativity) in more depth for a future blog post? I appreciate how you often can explain the technical in a way that gives non-physicists like me a better understanding of the science and issues involved.

    ReplyDelete
  27. The simulation is analog not digital. It does not calculate. Its size is not strongly bound to be larger than its construct's size. The universe is its own simulation! There's little employment to be had here.

    What simulates the simulation? God (retrograde intellectual diversity). Give generously to your local tax-exempt sales outlets.

    ReplyDelete
  28. Penrose is a mystic. I always have trouble taking him seriously.

    I've always liked Bill Gosper's approach in HAKMEM from 1972. He considers the various ways that a computer program can tell the number system of the machine it is running on. Back then, not all computers were twos-complement. There were ones-complement and a variety of decimal machines. He then raised the simulation issue:

    "By this strategy, consider the universe, or, more precisely, algebra: Let X = the sum of many powers of 2 = ...111111 (base 2). Now add X to itself: X + X = ...111110. Thus, 2X = X - 1, so X = -1. Therefore algebra is run on a machine (the universe) that is two's-complement."

    To capture the spirit of the times: "If arithmetic overflow is a fatal error, some fascist pig with a read-only mind is trying to enforce machine independence." Ah, the 1970s.

    ReplyDelete
  29. I don't disagree with the thrust of your argument, but one of your statements strikes me as unlikely, specifically "More likely, we will see a future in which rich nations can afford raising one or two artificial consciousnesses". This really reminds me of the early days of computing where many thought there'd only be a need for a handful of computers or that "640k ought to be enough for anybody."

    ReplyDelete
  30. Dev,

    Well, please allow me a little fun, I sometimes like to imagine what the future might look like. It's not that I think it'll stay that way forever. But keep in mind that in the early days indeed there were only a handful of computers. And they were huge. And not everyone got to book time on them. So it seems likely to me we'll pass through a similar state with quantum computers.

    ReplyDelete
  31. I see no reason to assume the "simulators" would go to extraordinary lengths to hide the truth of the simulation from us. So what if we discover it? Perhaps that's the point? Maybe that's when we "win the game"?

    Who knows? Maybe it's just experimental cosmology.

    What I find compelling about the hypothesis is the Bayesian argument of 'If we could simulate a universe, then so could someone else, and therefore it's more likely than not we're already in one (and maybe they are too!).' Also, if there turns out to be no TOE, then a simulation run on piecewise, discrete functions seems a perfectly compelling hypothesis. There being of course, no evidence to suggest a "theory of everything" exists, and much evidence to suggest it doesn't.

    PS It's trivially easy to hide "the truth" from scientists if you're God, just throw the tiniest bit of noise into something and subtle effects are swept right under the rug. Good luck with your 5-sigma if God's against you.

    ReplyDelete
  32. Sometimes we read that QM is like an 'operating system' (OS). I tend to agree with that. A question then arises. If QM is a sort of OS (superpositions, quantum darwinism, quantum non-separability, quantum contextuality, all those famous quantum principles, etc., are then routines or subroutines of that OS?) can we say that QM can ***not*** be, itself, a 'simulation'? It seems so, to me. (I remember that Einstein asked: Is the Old One [or the Great Simulator?] ***free*** to choose the physical laws?)

    ReplyDelete
  33. Hi Sabine,

    thanks for the post; I guess the laws of nature enabling a computer to simulate our universe would be very different.

    Why would this law be more natural than the ones we believe we know? If not, no interest.

    J.

    ReplyDelete
  34. Serafino,

    The OS metaphor is doubly wrong. First, QM is Turing complete and it's compares much more to a programming language than to an OS. Second, an OS suggests that there is something more fundamental and necessary to the QM existence and this proposition is false. The Turing machine demonstrates that computability emerges with a minimum of components, the QM does not need to be simulated, its simple existence is enough to generate the whole computability.

    ReplyDelete
  35. The simulation notion conceals professional envy. As a life-long programmer, I relate, even if I don't sympathize. Compared to today's virtual worlds we find the universe doesn't hang, doesn't crash, doesn't run out of resources, and doesn't need maintenance. There are no race conditions, no corrupt variables, no unintended side-effects. But this is like comparing a photo of a tree to a tree, it's not an analogy that flows in the other direction.

    The term "artificial intelligence" should be abandoned. When we create intelligence there will be nothing artificial about it. And there is no special reason to believe that we'll properly recognize it or fully understand it. Consider our failure to communicate with other intellectually advanced species, like whales and elephants. We do it on our terms, not theirs, despite decades of study.

    ReplyDelete
  36. I enjoyed the post, and don't like the simulation hypothesis either, but (like another commenter above, but independently) would very much like to hear more about this: "Indeed, there are good reasons to believe it’s not possible. The idea that our universe is discretized clashes with observations because it runs into conflict with special relativity."

    I had concluded (again as several commenters above) that any such simulation would have to be done in some higher-level universe with higher computational capacities, which takes it out of the realm of our science - however, I will say this: it is the first instance of a god-hypothesis (god as the entities in control of the simulation) which makes any sense to me. For example, a miracle (something not possible for the simulation's code to produce) could be done by the equivalent of hex-editing the data.

    As for Penrose's argument, mentioned above, here's an algorithm he seems not to have considered:

    1) Try something, even randomly, such as Edison's search for a practical light-bulb filament (not the first feasible thing, but the best thing he ever found in years of searching, was bamboo fiber).
    2) If it doesn't work, try something else.
    3) If it does work, write it down so future generations will know about it and go on to improve it.

    That's how blind nature produced Penrose. It is also similar to the way Dr. Bee found some equation-solutions mentioned in her previous post.

    ReplyDelete
  37. Sabine, four things:

    1. Physics is a mathematical that hopes to predict how the universe will behave on every level. In other words, it *encodes* the universe into algorithms. Our brains also encode what we perceive (differently). Language does the same (differently). We don't perceive raw reality, we perceive multiple, divergent encodings.

    One of these coding systems is maths. But we don't know how the universe *is*, we only know how we perceive it. We codify inputs into things that mean something to us.

    I am not a solipsist, mainly because I am not smart enough to understand how others codify reality. I can't experience how a physicist sees the curls and spirals that denote the decay of a charm quark; I can't experience the sensorium of a frog. But I can see these things happen. Therefore I am not alone.

    I can only code (and therefore understand) things that are within the boundaries of my own physical, experiential limits. (These limits grow with time and experience.) The fact that I can see that there is a larger 'super-set' of coding systems - and that there are possibly *super*-super-sets beyond - is proof, to me, that there is no way to directly apprehend reality.

    We can model reality's behaviour, somewhat, but we can't *experience* it in the raw.

    So, we absolutely *do* live in a simulated universe. The question is, is it just a product of our moment-by-moment encoding, or is there some larger principle at work?

    2. Sabine, you are quite right to say that it's mathematically impossible for a small fraction of the universe to be capable of perfectly simulating the entire universe it is part of - or, by extension, for that simulation to also contain another perfect simulation... But, like it or not, and however fuzzy the quantum edges might be, we do live in a granular universe. Our universe is not infinitely divisible.

    So, what if the 'simulation' that we live in is actually more granular than the universe that created the simulation? What if any simulation *we* make is more granular than our own universe?

    The paradox about universes-within-universes disappears. I haven't done any maths on this; but I assume you'd end up with a finite integral, even though there is an asymptotic infinity.

    3. You make assumptions about *why* some entity would run a simulation: that it is in order to receive some expected or unexpected output - a 'result'. This is anthropomorphic, and may not be the case. I personally can't imagine any other reason; but then I am human, and my thinking is anthropomorphic by definition. It doesn't mean such reasons don't exist.

    Maybe some meta-entity wants to know just how granular a simulation can become before it ceases to have meaning (on its layer of reality).

    Maybe Boltzmann brains are real (in some finer-grained universe), and the 'brain' happened to emerge with a coding system that simulates the universe we experience. (If it didn't, we wouldn't be here.)

    4. Why assume that the physics of our universe are the same physics as the universe running the simulation? Or even that the simulator-universe si part of our larger meta-verse? It could be something else entirely.


    One other observation, from Iain M Banks's novel "Matter". The novel is an exploration of what a universal simulation might look like, and how we can be sure that what we experience is real. One character points out that the best argument *against* our universe being simulated is that we suffer. We experience (and inflict) pain, mortality, horror, grief.

    What conceivable programmer could be so cruel?

    Then again, we are talking by definition about things we cannot conceive.

    Thanks Sabine, as always,

    Richard BW

    ReplyDelete
  38. Bee
    Your truth is your perception of reality interpreted by your human experience.
    In a nutshell this means that every human brain is wired differently. This would require a simulation for not only every human on planet earth, but also every mammal and possibly every living thing.
    david z

    ReplyDelete
  39. Hi Sabine,

    You are looking at it from very limited perspective. Had you ever thought of reading about Rene Descartes? Everything comes down to energy. Even a thought and idea is made of energy. And energy can be neither created nor destroyed; rather; it transforms from one form to another.

    ReplyDelete
  40. I think one should separate out two kinds of simulation arguments: anthropocentric and non-anthropocentric. The first sort, I think, is, while not entirely impossible, certainly implausible for a number of reasons you point out. Unfortunately, people tend to make their arguments for what we should do on this basis - "humans should be interesting" and so on. The second sort, I'll come back to.

    There are also two ways to take simulation arguments: as science and as philosophy. The former is concerned with predictions from the hypothesis; the latter is concerned with arguments for or against the hypothesis as "true" (metaphysically). Many scientists, when discussing the simulation hypothesis, are doing so not as scientists, but as (typically naive) philosophers, dealing vaguely with ideas rather than actual concrete predictions (although there are a few who make predictions, whom you point out and whom I was previously unaware of).

    That said: let's take a non-anthropogenic simulation hypothesis. I don't think this is entirely unreasonable; it just takes simulating a sufficiently complex universe. However, since we have no idea what people who simulate us are simulating (or what technology they're using to do it), that seems to tell us nothing new scientifically. Insofar as the nested-simulation-argument is valid, the simulation hypothesis makes no predictions.

    My own issue with the argument in principle: you can then ask "what counts as a simulation." Sure, an actual computer simulation does - but what about a by-hand calculation of the same thing? What about mathematically solving the differential equations for the same phenomena, providing the same solution? What about just thinking about the solution? If a teacher teaches his students how to solve this differential equation, does that spawn a bunch of new universes? I think if you take this to its logical conclusion (combined with something like mathematical Platonism - things turn out differently if you think the math has to actually be discovered to exist), you find that universes can only be the laws themselves in some abstract sense, and you more-or-less end up being Max Tegmark. Not that there's anything wrong with that, but again, I think that ends up being unhelpful and unpredictive metaphysics rather than anything scientific.

    ReplyDelete
  41. We do live in our own brain's simulation of a virtual world based on real world information. What we experience is not the real world but the virtual world, albeit event in the two worlds are highly correlated (unless you suffer from schizophrenia). You can do simple experiments to verify this, e.g. optical illusions like this one

    https://twistedsifter.files.wordpress.com/2012/05/optical-illusion-same-gray-color-thumb-over-middle.jpg?w=800&h=600

    if you obscure the boundary between the two squares with a finger, then both squares become equally dark. Or take the McGurk Effect:

    https://www.youtube.com/watch?v=G-lN8vWm3m0

    ReplyDelete
  42. It helps also if you remember that simulating a brain doesn't mean simulating something that always gets right answers. Most conscious animals we know use a biologically implemented Bayesian scheme that only indirectly requires any QM effects. It's a pretty good scheme, but it isn't always right and isn't particularly "quantum".

    ReplyDelete
  43. From what I understand the simulation hypothesis is supported by mathematics but is not science. It's not falsifiable from my understanding, it doesn't make any predictions which can yield a true or false discovery, it does not reveal anything new about the universe. It does imply a multiverse in my opinion.

    The simulation hypothesis suffers from the same problem that the problem of other minds suffers from. It cannot be tested, it's not science, but is a thought experiment. You can create a proof or come up with probabilities but if it doesn't produce something testable by scientists, physicists in particular, then what good is it?

    So we live in a simulation? So what?

    ReplyDelete
  44. While evidently not viable as a scientific theory, the simulation hypothesis would make a nice sci-fi background plot for a Star Trek episode.

    ReplyDelete
  45. @mlmcl... "Consider our failure to communicate with other intellectually advanced species, like whales and elephants. We do it on our terms, not theirs."

    If cetaceans broadcast (encoded) sonar images, we are fools for assuming our linear versus their volumetric language. Perhaps lines of physical theory should be 2D+ε holographic projections (photographic thick emulsion versus binary optics embossed film) re Contact (1997) and the primer. Holograms do not calculate. Lenses do not calculate, but they lose phase information.

    ReplyDelete
  46. Nicolas [9:11 AM, March 16, 2017]. I can agree. QM compares more to a programming language (or a syntax or, maybe, a many-valued language like the Aymara language) than to an operating system. I also think that QM does not need to be simulated. I'm in trouble when I think about the very meaning of computation, in the QM context. What is computed? Information? What is information? How is this information Turing computable? Via wavefunctions? Another interesting subject is, of course, the Turing-Church-Deutsch principle, and related consequences. S.

    ReplyDelete
  47. "The idea that our universe is discretized clashes with observations because it runs into conflct with special relativity." If nature is a multiverse that needs to be explained in terms of a Fredkin-Wolfram network underlying the Planck scale, then all arguments involving energy, spacetime, and/or quantum information might be wrong unless justified by some approximation that uses Fredkin-Wolfram information. Google "einstein's field equations: 3 criticisms" for my viewpoint.

    ReplyDelete
  48. I never could tell the difference between 'the universe is a computer simulation' and 'there exists a god, and perhaps that god is even similar to one in one of our religions'. As a scientist I don't think it is an interesting hypothesis and as a practicing religionist, after a little thought, I concluded that it also wasn't relevant to my relationship with God. That doesn't mean that the hypothesis doesn't imply anything interesting to theologians or philosophers, for example it might imply that god (or that which is doing the simulation, to use less loaded language) wasn't perfectly omniscient/omnipotent.

    ReplyDelete
  49. The whole notion of creating an "artificial consciousness" in a simulation, including encoding a "sense of agency" and self-awareness, seems outlandish.

    Many very smart physical scientists & engineers, who often have a reductionist world-view, seem to significantly misunderstand & underestimate the problem. Ray Kurzweil's "singularity" mumbo-jumbo also falls into that trap.

    Consider the model organism, the nematode c.elegans. Every worm has an essentially identical "brain" of about 300 neurons. I believe its neural connections -- its "connectome" -- have been completely mapped.

    Yet, how the connectome leads to the worm's most basic behaviours is poorly understood, eg see:
    http://www.cell.com/current-biology/abstract/S0960-9822%2805%2900940-1?_returnURL=http%3A%2F%2Flinkinghub.elsevier.com%2Fretrieve%2Fpii%2FS0960982205009401%3Fshowall%3Dtrue

    IMO a more likely scenario for "artificial consciousness" is creating a "chinese room" type of simulation. It might very well pass any Turing test, but it's still just an unconscious "chinese room".

    -- TomH

    ReplyDelete
  50. > If you try to build the universe from classical bits, you won’t get quantum effects, so forget about this – it doesn’t work. This might be somebody’s universe, maybe, but not ours. You either have to overthrow quantum mechanics (good luck), or you have to use qubits.

    Actually, if you have exponential classical resources available, there's no problem. This was proved in one of the first papers on quantum computation: http://epubs.siam.org/doi/abs/10.1137/S0097539796300921

    >But how does the programmer notice a simulated mind is about to notice contradictions and how does he or she manage to quickly fix the problem?

    This also seems like a non-problem. If it's a simulation, it could simply be restored from an earlier state as necessary, like save-scumming.

    >Besides, if the reason you build an artificial intelligences is consultation, making them believe reality is not what it seems is about the last thing you’d want.

    However, I'm totally in agreement that the motivations of the programmers and hardware-owners are so bizarre and inexplicable as to throw the whole enterprise into serious doubt. Of what possible value is our universe to anyone who doesn't live here?

    ReplyDelete
  51. Steve,

    Yes, first statement was very sloppy, sorry :/ Quantum mechanics should be possible as long as you have some kind of non-locality (in a suitable 'space-time'), it's quantum field theory that nobody knows how to do from classical bits. Hence my remark 'good luck': it might not be impossible, but nobody knows how to do it, and it will take some work to convince anyone it can be done. (The issue of locality is a thorny one. I am not at all sure it's even possible to replace the actual 'space-time' with a made-up space-time since locality is basically what defines the thing which we call 'space-time'.)

    Second, I don't see how this helps - you'll still have to do the calculation which is what you were trying to avoid in the first place.

    Either way, the message I was trying to get across is that if one wants to claim we live in a computer simulation one has to show it's possible to reproduce our observations this way. That's not easy, and vague words won't do.

    ReplyDelete
  52. Whenever somebody espouses the simulation hypothesis to me I have to remind myself to be tolerant of religion. It is after all no more or less absurd than the idea that some guy thousands of years ago was nailed to a piece of wood to wash away our sins (whatever that is supposed to mean).

    ReplyDelete
  53. @Henning Dekant
    " It is after all no more or less absurd than the idea that some guy thousands of years ago was nailed to a piece of wood to wash away our sins "

    Human sacrifice was quite common throughout human history. Nothing singular about this story I think. Just that this particular way of torture was a common way to execute criminals.

    ReplyDelete
  54. Two random comments:-

    1) For me the simulation idea falls down with "combinatorial explosion", which Daniel Dennett talked about in Consciousness Explained when discussing the Brain-in-a-Vat idea. Something like what you said about the choices an intelligent mind is going to make - a simulation would have to "anticipate" all the possible choices, and the possible results of those choices, etc., etc., ad infinitum.

    2) Re. AI, I think that even with the great strides machine learning has been taking recently, it's not going to amount to actual intelligence: that will require different AIs talking to each other and making their own world together. IOW, intelligence is partly a function of sociality, most of the most intelligent creates are intensely social animals (with the odd exception of the octopus).

    ReplyDelete
  55. It's not clear to me whether the result in the paper cited by Steve:

    http://epubs.siam.org/doi/abs/10.1137/S0097539796300921

    applies here. Suppose we just do a brute force simulation of QFT, just put everything on a lattice and compute the time evolution of the wave-functional. Then the classical simulation will yield a MWI-like evolution, you'll end up with different sectors that have a different classical history that you can assign probabilities to using Born's rule. But we're interested in the inside view, and at that level Born's rule doesn't apply because all sectors end up being simulated, an observer has equal probability to find him/herself in any of the sectors where he/she is present regardless of the complex numbers referring to amplitudes that are supposed to yield the probabilities for the sectors.

    ReplyDelete
  56. You want our familiar length scales like the size of atoms to be much larger than the lattice spacing. One should therefore tune the bare couplings of the model defined on the lattice such that our familiar physics appears at scales much larger than the lattice spacing. The lattice artifacts lead to irrelevant operators, so they then become invisible at the scales we can probe. This can be done by using a model that has a critical fixed point and choosing the bare parameters such that the RG flow will let you hover near that fixed point for a large number of RG steps before you veer off toward the Standard Model.

    ReplyDelete
  57. Count,

    As you have noted yourself, you can do it up to a limited precision, highlighting the problem I pointed out: How does the programmer in advance what precision is about to be tested and if the precision limit is reached, how does he-or-she know what data to fill in?

    ReplyDelete
  58. Sabine,

    Of course, I don't believe it's true, but I guess that a simulation on a lattice would not allow field configurations to arise that correspond an observer breaching the limits of the simulation.

    Another problem is that the simulation actually doesn't seem to matter. Suppose that you simulate the early universe just after the Big Bang for a few seconds and then you stop. We then don't seem to appear in that simulation. However, the universe as it exists today is obtained by applying the time evolution operator to the early universe. Therefore, that simulation of the early universe does contain us in a scrambled way. This suggest to me that the simulation wasn't necessary at all, leading me to Tegmark's mathematical multiverse.

    ReplyDelete
  59. Sabine H.>> I think most people are much smarter than many self-declared intellectuals like to admit. Most readers will instead correctly conclude that today’s intelligentsia is full of shit. - I agree 100% with this part, but not the rest. Thanks for your interesting post!

    ReplyDelete
  60. Hi Sabine, I feel you haven't fully grasped the implications of the theory that 'reality is information' (or reality is computation).

    It's *not* saying that 'reality is a simulation'. No, it's saying something much more radical than that - it's saying that there is literally no difference between 'the real world' and 'a simulation'. It's saying that the whole notion of a 'real world' is meaningless - it's like the aesther and can be dispensed it. The idea is that there's no hardware at all, no 'base level' exists - it's all 'software'.

    This can work by allowing the quantifier 'real' to be continuous instead of binary. Instead of 'real' as being a binary yes/no, we could think of 'real' as a continuous, like the brightness of a light bulb. Then the idea is that there are only *degrees* of reality, things are more or less real, but no 'base level' of reality is needed.

    Some questions for you: Is a simulated hurricane a 'real' hurricane. Answer: To some degree yes. Is a simulated you, really you? Answer, again, to some degree yes, up to and including 100%.

    Is the 'simulation' of a thing actually drawing that thing into existence, by making it more 'real'?

    And now here's the really huge kicker for you, the 'astonishing thought': Is the creation of the universe really completed yet, or is it still going on?

    Think: what happens when one part of the universe simulates another part? (Remember: 'real' can be a matter of degree, and the simulation of a thing can literally summon that thing).

    ReplyDelete
  61. zarzuelazen,

    I think you haven't fully grasped what this blogpost is about.

    ReplyDelete
  62. Actually this issue I think is misunderstood. We do not need to simulate the whole universe to prove it. We only need the correct form of the theory of nature to calculate, let's say, interactions of 100 atoms to the experimental precision(all aspects). It is clear that present day theories are lacking, but it is very clear that today's theories are also very close. So obviously we are almost there to prove the simulation hypothesis, at least that it is possible(maybe by an advanced civilization that has solved the problems of the human ones).

    ReplyDelete
  63. To paraphrase G. Orwell, "They must be real intellectuals, no ordinary person would believe such nonsense."

    Quis simuladiet ipsos simulades?

    Did J. A. Wheeler know bit from Shinola?

    So many other interesting problems: dark matter, matter-antimatter asymmetry, measurement problem, dark energy, neutrino oscillations, etc.

    ReplyDelete
  64. zarzuelazen,

    in any case HW or SW or both, what does that change? That you and I can refer reality to some macroscopic category inherited from our immediate environment. Why would there be even the beginning of a similarity between that dichotomy and reality?

    If only information and no substrate, then information is active and interacts: I do not name this information.

    J.

    ReplyDelete
  65. qsa,

    if you think we're close to simulating the presently accepted fundamental laws of nature, I recommend you read my post again

    ReplyDelete
  66. Sabine,

    I have read the article(many of the same arguments elsewhere) and many of the comments. It would help if your objections are spelled out more explicitly. Thanks.

    ReplyDelete
  67. The problem is a curriculum problem, computer scientists have to go to quantum computing courses (at least as an option) and physicists have to take courses in the theory of computation (at least as an option). It is indispensable that the notions of computable numbers, formal incompleteness, automatons, complexity of the calculation be well grasped by physicists. This to avoid reinventing the wheel and to marvel at what is obvious to the other.

    ReplyDelete
  68. A nice post: thank you. One small correction: I've asked Nick several times over the years what credence he personally assigns to our living in an ancestor-simulation. He's never reported a figure higher than 20%. That's rather different from believing that if posthuman superintelligence runs ancestor-simulations, then the principle of mediocrity dictates we're probably one of them – which is Nick's view.

    ReplyDelete
  69. Mmm. I've only recently started to read your (most enjoyable) blog as part of my self education in science, and so have never heard of the simulation hypothesis before today.

    You wrapped up your piece with:

    "Most readers will instead correctly conclude that today’s intelligencia is full of shit. And I can’t even blame them for it."

    And I was relieved that you had written that, Sabine, because as I was getting to the end of the blog post the thought that was pounding in my head was, "Do people actually get PAID for thinking up this **##* stuff?!"

    I'm far too well mannered to have written a comment saying what you yourself said - but it was marvellous to have it said for me!

    ReplyDelete
  70. @qsa, "interactions of 100 atoms to the experimental precision"

    Orthorhombic sulfur: space group Fddd (#70); a = 10.4646 Å, b = 12.8660 Å, c = 24.4860 Å; α,β,γ = 90°, has 128 atoms in its unit cell (DOI: 10.1107/S0108270187088152). You might need more than 100 atoms given that sphere close packing is 74% occupancy and sulfur manages only 17%.

    ReplyDelete
  71. Im a geologist.

    On all of these issues I use a geologic timescale. We are still motes on a dot. In a thousanf years? A million? A billion?

    We dont know what consciousness is. We don't yet have full AI. We still haven't had any interaction with some alien intelligence. The bottom line is we are primitive creatures. Proposing some type of simulation is just another level that our brains can 'sort of' get around...the same with multiverses, parallel universes, etc. There is ZERO evidence for it yet its tossed out as a possibility. Its akin to
    Creationism...no less legitimate yet no more absurd. Nothing in the physical properties of the Universe suggest a god or a simulated existence. It needs not be refuted as there is nothing to refute.

    The next hundred years are going to offer some exponentisl advancements in technology and our understanding of existence. We may discover that 'reality' is more mind blowing than the possibilities we toss out today like 'simulation'. Today we our frustrated by our physics and impatient but the legitimate answesr will still be evidence based.

    ReplyDelete
  72. qsa,

    I explained in my post that your argument 'we do not need to simulate the whole universe' is an unproved and non-trivial assertion, so is your belief that we can reproduce the presently accepted fundamental laws of nature (the standard model and general relativity) on computers. We can't. We can only approximate them. I don't know why you expect me to repeat this and I'm not interested in repeating it yet again.

    ReplyDelete
  73. The notion that we have solved problems that can't be solved by algorithms seems suspect. What counts as a "solution?" As long as it can be described and verified in finite time, it can be found (after a very long time) by a simple trial and error algorithm. And if it can't be described and verified in finite time, then how can we claim to have solved it?

    Proving a problem unsolvable isn't the same thing as solving it! The so-called halting problem is a great example. The name is misused; is the HP finding a general program for determining halting behavior, or is the HP determining whether such a program can be found? The latter is solvable and solved; the answer is no, which is why the former is not and cannot be solved. Claims that the HP is an unsolvable solved problem trade on confusion between these two definitions.

    That being said, Penrose is a pretty brilliant mathematician, and I think this is evidence that these things are hard rather than that some people are stupid or malicious for believing false or confused statements.

    This is actually why I take Bostrom seriously! I don't necessarily think his argument is right, but then I'm not sure HE does. The point of pushing an argument like this is to see where and how it fails. I'd argue that Sabine's criticism here only applies if the argument can't fail — if it can't, then it's "not even wrong" and so frivolous. But I think it can fail, and in interesting ways that can teach us something. The fact that it can't be rejected based on physical evidence doesn't mean it can't be rejected, nor that such a rejection would invalidate the exercise. This is how philosophy (and math, I should add!) makes progress. It's just as hard and just as important as physics, though I don't necessarily expect physicists to agree!

    My favorite analogy is with go (or chess if you prefer). There are some moves that you don't ever see in a pro game because they end badly. But knowing that they end badly requires playing them out! That's what I think Bostrom is doing; or at least that's why I think what he's doing is valuable, even if it turns out he's a confused true believer.

    I'd be happier if this post engaged with the actual probabilistic argument he makes. It's not trivial, it can be justified by certain assumptions, and those assumptions are accepted in some other contexts. The question is, why should we reject them in this context? I feel that Sabine has given only half an answer here, and has not done anything to link that answer, in a careful way, to Bostrom's actual claims. As someone who takes the call to quantify humanistic and philosophical arguments very seriously, I'm slightly bothered by this unwillingness to meet half-way.

    ReplyDelete
  74. David,

    I've read your comment 5 times, but failed to make sense of it, sorry. He believes we live in a simulation with 20% probability or he believes we probably live in one of them (assuming that 'probably' means 'with almost 100%')? Your comment seems to say both.

    ReplyDelete
  75. Scott,

    I think you misunderstood the message I tried to get across. It's not that I think it's uninteresting, or that nothing can be learned from it, or that one shouldn't try to estimate the probabilities. I'm saying if you want to do that, you have to take into account the physical laws that we have confirmed to high precision. That's a non-trivial requirement, and without demonstrating that you can actually reproduce all our observations the simulation hypotheses is merely fiction. Best,

    B.

    ReplyDelete
  76. What's wrong with treating the universe as a computer?

    I mean, if you argued against it by saying 'So where is that computer? Inside another universe? Is that a computer? Computers all the way down?', that's a valid argument against it.

    But otherwise if a universe works by fixed rules and does not deviate from them, what's wrong with calling it a computer? Is the problem that 'computer' seems too intentful a word?

    ReplyDelete
  77. Sabine, apologies for the ambiguity. I just checked the Simulation Argument FAQ (last updated 2011). Nick explicitly states that he assigns a probability to the simulation hypothesis of something in the 20% region. This is quite consistent with believing that _if_ posthuman superintelligence runs full-blown ancestor simulations, then we probably inhabit one of them. Maybe such simulations will prove technically infeasible; maybe humans will shortly go extinct; maybe any posthuman superintelligence will find ancestor-simulations too unethical or uninteresting to run. Maybe (and this would be one of my doubts, not Nick's) phenomenally bound subjects of experience can't arise at different levels of computational abstraction. Either way, Nick is much more cautious than some of his popularisers. Compare Elon Musk's recent claim that the probability we live in basement reality is “one in billions”.

    ReplyDelete
  78. David,

    Ok, I see. My biggest issue with Nick isn't the number but that I think it would be wiser to not mix up the simulation hypothesis with existential risks that actually have a sound footing in science.

    ReplyDelete
  79. Cario,

    Why don't you read what I explained in my post: It isn't compatible with what we know about the laws of nature. Unless you can prove that it can be made compatible, it's merely a mildly interesting tale.

    ReplyDelete
  80. Cairo...is the Universe a computer?

    Maybe yes. maybe no. But evidence is needed. We have none.

    We don't know what existence is so we don't know if it follows some rational process. Our senses perceive fragments of reality that our minds are capable of detecting. Our math and physics are akin to watching a baseball game. Everything seems rational to the fans watching. Patterns are just accepted and some theory might be developed about how the number of strikes relate to the number of outs, etc. However, when we stand back and ask someone who has never seen a baseball game if the patterns represent reality he would go 'huh?...it has nothing to do wih gathering coconuts'.

    The computer model, simulation, etc. 'assumes' a greater grasp on existence than what we have. Its akin to the ancients assuming that some god is responsible for the annual seasons. We are are still clueless to the extent of knowledge that we dont know. There may be a simulated existence, there may be a Sun god, the Universe may be a cimputer but 'so what?'. No evidence for any of these. Zero. We have finite knowledge and, like baseball, it may be irrelevent in understanding the bigger picture.

    ReplyDelete
  81. Sabine, obviously we might "live in a simulation". OTOH obviously you know what you're talking about. So, there's an apparent contradiction.

    Minimally, a virtual reality simulation has to reproduce my experiences - or, from your point of view, yours. Most people say their consciousness can't be simulated, but not you. So, surely you agree that you personally could be a "brain in a vat"? Unlikely, but doesn't violate any physical laws, right? Moving beyond solipsism, if it can do one human, it can do 7 billion. A few orders of magnitude extra sim power is no big deal. Now, let's consider some of your objections.

    From your previous post on the subject, "To avoid the inconsistencies, you’ll have to carry on all results for all future measurements that humans could possibly make, the problem being you don’t know which measurements they will make because you haven’t yet done the simulation." I guess you're talking about (to take simple example) a classic spacelike-separated Alice & Bob EPRB gedanken? The sim can easily provide random spin up/down results for A and B. But later, when results are compared, they must be consistent, with the right correlations. If that's what you mean, I claim it's no problem, happy to explain why.

    Not sure why you think sim has to look at "all future measurements" Alice and Bob might make. But the objection is easily disposed of. As far as we know there may be only one future path. The entire history of the universe may be pre-known: no calculation necessary, sim just looks it up in a large database.

    Then you say: "But there is a better way to test whether we live in a simulation: Build simulations ourselves, ... Eventually, the ... lowest level will find some strange artifacts. Something that is clearly not compatible with the laws of nature they have found so far and believed to be correct." This is no argument against sim hypothesis. Leaving aside some (rather important) issues, there's no reason this scenario couldn't happen. It may seem "crazy", but that's no argument.

    In this current post, you say "... there’s a trivial way in which the simulation hypothesis is correct: You could just interpret the presently accepted theories to mean that our universe computes the laws of nature. Then it’s tautologically true that we live in a computer simulation. It’s also a meaningless statement." Wrong, it's far from meaningless. If we really are brains in vats, fed by thick conduits carrying encoded qualia, generated by a physical computer of some sort, tended by insectoid alien programmers - that's non-trivial! If the computer happens to generate those inputs by solving Standard Model equations, so what?

    You assume sim must use the most advanced technology we ignorant naked apes can (sort of) understand at this stage of our development: digital quantum computers. But we have no idea what powerful computing resources might be available to us 100 years from now, much less a million. We can't constrain hypothetical alien programmers to our primitive techniques.

    Although you're aware that only human experience needs to be simulated, you seem to think in order to do that, simulation of the entire universe, at all scales, must ultimately be involved. As you show, with primitive digital quantum computers, that probably can't be done. But your assumptions are flawed.

    The solution to the "apparent contradiction" mentioned above may be: you're really not debating whether sim is possible, logical, or reasonable. Instead, you're investigating whether we can prove it or not: whether it's a scientific hypothesis, in the Popperian sense. But before going on I'll wait for response (if, indeed, you consider it worthwhile). BTW I don't "believe in" sim: although there's absolutely no evidence against it, there's not much for it either. Thanks for your time.

    ReplyDelete
  82. George,

    "Although you're aware that only human experience needs to be simulated, you seem to think in order to do that, simulation of the entire universe, at all scales, must ultimately be involved. "

    What I am saying is, if you believe you can get away with not simulating parts of the universe when some of your simulated consciousnesses - whose actions you can't predict - aren't trying to probe them, I want to see how you do this. Unless you demonstrate that, it's just bla-bla but nothing any scientist can take seriously. The idea that you can somehow produce an explanation for all our observations that does not require the laws of nature to be how we have extracted them today is an extraordinary claim and nobody should accept it without a very solid backing. Best,

    B.

    ReplyDelete
  83. Hi Bee,

    Let's assume the universe could be described with set of equations, and an initial condition. Then there (outside of our universe) can be a number of computers which can run the simulation and all of them would get the same result (assuming the functions are predeterministic).

    If the number of computers running this simulation is more than 1, could someone inside the simulation have means to check out how many computers there are runnig the simulation? I guess not.

    If a number of those computers are running the simulation in synchronization (so that you and me are writing excactly the same letters in same their-time), and then one by one those computers are shut down. Would we notice any difference? Even when the last computer is shut down? I believe not.

    What if the equations are such, that the simulation can be done in kind of frequency domain (as opposed to time domain). What would be the point in time (in the simulating computer's wolrd time) that I press this T-letter? It's an ill-defined question.

    If the frequency domain simulation was such that it could be distributed to several computers, which could run in parallel or totally in different times. How would it affect our capability of detecting the simulation? I'd say we couldn't sense it.

    What if someone invented the equations, and started simulation in frequency domain, but aborted it after running a small part of it. Would we exist, and could we feel the simulation is incomplete? I say no, even if the small part is like one millionth of the whole simulation.

    And the logical next question is, is there a need to start the simulation for us to feel our own existence? Or is there a need to "invent" the equations in the first place, for our everyday feelings?

    BR, -Topi

    ReplyDelete
  84. Sabine, it's quite plausible the sim could predict everyone's actions. Indeed, the entire future (not just human actions) may be pre-determined. Isn't Block Universe the most popular model among physicists today? If spacetime is static, sim would just be running a pre-recorded script. IOW humans would have no free will. Why are you so sure that, on the contrary, they do? Unless you demonstrate that, it's just bla-bla but nothing any scientist can take seriously. I won't address other points now, since this one is rather important. BTW Of course a sim must follow the laws of nature as we understand them. They simply describe - to pretty good accuracy - what we observe, and that's exactly what the sim must reproduce. Thanks, George

    ReplyDelete
  85. George,

    Please read more carefully. That something is determined doesn't mean it can be predicted.

    That you accuse me of being sure that humans have free will is another demonstration how quickly people jump to mistaken conclusions about my presumed opinions.

    Actually the simulation doesn't have to follow the laws of nature that we know - another sloppy misreading of yours. It has to, erm, simulate them. And therein lies the problem. If you still haven't understood why that's nontrivial, please re-read my blogpost.

    ReplyDelete
  86. Sabine,

    Thank you for this. I've been saying this for a long time. But it sounds more convincing coming from you.

    ReplyDelete
  87. I think your argument is flawed. This explains why:

    http://www.smbc-comics.com/comic/2010-11-09

    Computing devices outside could be far more powerful than those inside. The simulation also need not run in real time or model the same universe. The simulated physics in a computer simulation containing sentient beings are the laws of physics as far as they are concerned. Also, WRT running in realtime, keep in mind that Turing completeness allows any language to simulate any other language, but makes no time guarantees.

    Your argument is weak because you are trying to discredit something that is unfalisible.

    ReplyDelete
  88. You're basing your opinion on what you know, and/or what you think you know. The fact that the computer you wrote this blog/rant on wouldn't be able to simulate the universe doesn't mean it is impossible for a computer that could to ever exist. The fact that something seems complicated to you doesn't mean it is complicated, period. Everything's relative. ;) The computer you wrote this blog/rant on is absolutely insane compared to computers 50 years ago. And in five years it'll be a piece of crap. Again, basing your opinion on the current capability of the computers we have now makes no sense. The fact that we think the universe is complicated and that we don't know how it all works does not mean that it couldn't be simulated by a computer more advanced than computers we have now, programmed by "people" or AI more advanced than us.

    "If you try to build the universe from classical bits, you won’t get quantum effects, so forget about this – it doesn’t work." Unless you program it to do so. You don't seem to be too familiar with how computers work, frankly. Basically, your argument is similar to saying we can't have fractional numbers in computers because it is only 1s and 0s. Just because you don't know how to program something doesn't mean nobody else could be able to figure out how to program a solution to that problem either. There is absolutely nothing about the nature of quantum mechanics that would be restricted by a binary computer. And there's absolutely nothing saying a simulation has to be running on a binary computer as we know them, either. Sorry, but basically your whole rant is summed up by "I don't know computers very well, so it can't be computers." Makes no sense.

    ReplyDelete
  89. Srk9,

    So, erm, what is it then? My argument is flawed or it is weak? As to your first "criticism", you seem not to have read anything I wrote, you just produce further "it might be so or so". Write it down, demonstrate that you can reproduce the standard model and general relativity, then come back.

    As to your second comment, I am not trying to discredit it, I just did.

    ReplyDelete
  90. Shorty,

    So what do you recommend? That I base my opinion on what I don't know? Like the people who believe that the universe can be simulated on a computer but who don't bother to find out whether the laws of nature actually can be simulated?

    Having said that, you are assigning opinions to me that I don't hold. I'm not saying it's not possible that some day some being somewhere might be able to simulate a reality like the one we experience. I am saying that this claim isn't supported by what we presently know. It's speculation. It's a mildly interesting fiction. But not science.

    ReplyDelete
  91. Anything can be simulated. You need to stop thinking that something can't be simulated because your Dell is a binary computer. There is a reason I quoted that specific sentence of yours. Because it is wrong. Absolutely nothing about a binary computer restricts it from simulating the probabalistic location of an electron. Why you would think it does is beyond me. Like I said, you don't seem to understand how computers and/or programming works. Your opinion is most certainly based on that fact. But even learning a tiny little bit about computer programming would show you how that opinion is not a very good one.

    ReplyDelete
  92. You say "if something is determined doesn't mean it can be predicted". Well, no, but it sure makes it a lot easier! Look, forget that superfluous word "determined". The question is: is it reasonably possible that sim could predict human actions? I re-read your blogpost, as requested. Here's the only relevant statement: "If the programmer could predict in advance what the brain will investigate next, it would be pointless to run the simulation to begin with." Fine, I can't think of a better argument. But surely we don't want to try to guess the hypothetical motives of a hypothetical programmer?

    Re. "accuse" and "sloppy" - fortunately I deal with young people all the time, and know how easily you guys take offense. (I was much, much worse at your age.) You were saying human actions can't be predicted. Incorrectly, but reasonably, I equated that with free will. Apparently you don't believe in that. It would be best to merely say so - don't you agree? Ok, forget free will. No problem. But then, you can't use it to deny the prediction hypothesis.

    Conclusion: as far as we know, it is indeed possible that sim can predict human actions. If not, please give some logical justification for your view. At the moment I have no idea what it would be.

    You say "Actually the simulation doesn't have to follow the laws of nature that we know - another sloppy misreading of yours." No, it's not sloppy misreading, but sloppy language. I meant that its results must "follow" those laws, not that its unknown physical mechanism must obey the laws of our universe (which is obviously wrong).

    You say "If you still haven't understood why that's nontrivial [simulating the laws of the universe], please re-read my blogpost." Ok, I did. Of course it's nontrivial for us, with our primitive computers, but sim could be (let's say) 10^100 times more powerful. It can merely solve those exact laws - not for the whole universe, just for the observations made by humans. You seem to think that implies space and/or time must be discrete. No, it doesn't - as Aristotle pointed out long ago. Happy to explain his (correct) reasoning, if you wish.

    Certainly, you have some knowledge about this issue I don't. Let me admit that right up front. OTOH there may be things I know that you don't. That's conceivable, isn't it? At my age, the goal of conversation is not to prove who's got the bigger {whatever}, but to learn - and, to teach. To share information, achieving a consensus opinion if possible. Doesn't that make sense?

    ReplyDelete
  93. Sabine,

    I googled you and found some really awful things from Lubos Motl. I see also you're arguing with Scott Aaronson. Finally, some on this blog are hassling you. Obviously you're very busy and have more important irons in the fire than me. It's understandable that your patience wears thin. Please don't feel obligated to address my points, it's not important. Although I don't happen to agree, I admire your taking a stance and fighting it out. Someday when you have the time I'd like to continue our conversation. Good luck, and best wishes ...

    George Rush

    ReplyDelete
  94. Dear Sabine,

    Your post raises a multitude of very different and controversial points. This makes it very exciting (and the comments heated). Thank you for the brainfood!


    1. Computational vs. computable:

    A lot of readers point out that any physical theory is a computational theory, i.e. that the standard model and general relativity describe the universe as computational, even though it might not be (Turing) computable. A quantum computational universe would arguably be in BQP, i.e. it is effectively computable but not efficiently. The universe could also be hypercomputational (for instance because it is continuous or even acausal), but while that would be bad news for current computer science, it means that it is possible to physically implement hypercomputers or acausal computers (and universe-simulations), no?

    If the universe is not computational, what else would it be? Is it possible to express a noncomputational theory of the universe in a formal language, or to even think about it?

    2. Must the universe be computable?

    Digital physics seems to be a minority position, and fraught with difficulties. I think it is great that you take a strong stance and argue that it is impossible in principle, for instance because Lorentz symmetry can in principle not result from discrete operations. I am not a physicist, but it seems to me that this is a highly surprising result and not trivial to prove, regardless of the difficulties to get discrete models to work. Until such a proof exists, it seems to me that rejecting digital physics may be justified only pragmatically?

    3. Methodology of philosophy:

    In the face of the absence of a generally accepted proof that the universe is uncomputable, and a considerable community that thinks it is: is it acceptable to consider the implications of a computable model without having solved the problem of deriving the standard model in that framework? This seems to be unfair to ask of a philosopher.

    4. Simulation vs. computation:

    If the universe is computable, it does not follow that it is a simulation. Even the inverse is not true, i.e. if the universe implements hypercomputational capabilities, then it is not clear why no hypercomputer running a universe simulation can be built.

    But if we grant that the universe is computational or even computable, it is still extremely unlikely to be a simulation. I think here you have the strongest point to make against Bostrom: if the universe is a simulation, it is probably not one that is run by a posthuman civilization to learn about its own evolutionary history, because generating a googillion galaxies for the sake of simulating part of a planetary surface for a few thousand years is implausible.

    5. Artificial intelligence:

    You write "It’s not that I believe it’s impossible to simulate a conscious mind with human-built ‘artificial’ networks – I don’t see why this should not be possible. I think, however, it is much harder than many future-optimists would like us to believe. Whatever the artificial brains will be made of, they won’t be any easier to copy and reproduce than human brains. They’ll be one-of-a-kind. They’ll be individuals."

    As a computer scientist, I find this baffling. If you build a neural network, even a very large one, what will make it hard to copy and reproduce it?

    ReplyDelete
  95. A lovely rant!

    P.S.: intelligencia -> intelligentsia

    ReplyDelete
  96. The simulation hypothesis seems feasible to me. BUT it's probably unfalsifiable...which means we probably shouldn't worry about it because it's up there with religion, the tooth fairy and other unfalsifiable things.

    I think I see things in the laws of physics that a computer programmer would have thought to be a good thing to do. Having a finite speed of light, for example, makes parallelism in the calculations much easier and provides for a natural "edge" to the observable universe without needing a "wall" or some other unlikely-seeming boundary. The Big Bang is another handy thing - it provides a finite limit to history and a simple way to populate the simulated universe with fun "stuff" without hard-coding it all.

    The fuzzy nature of things that happen at quantum scales just screams "round off error" to me - and although the things we know about what happens at those tiny scales don't really match the nature of the digital computers we've made - that doesn't make much difference. The laws of physics in the "parent" universe would have to be a lot different from ours in order for them to have the computing power they'd need. So all bets are off.

    But all of this is essentially unfalsifiable. So we should make a note of an interesting hypothesis - and get on with figuring out what we can about the universe as we see it.

    If evidence ever does come to light to prove that we're in a simulation - then we can start to be concerned about it.

    ReplyDelete
  97. Sabine

    No need to be annoyed, the idea (our universe is a simulation) is easily dismissed.

    The idea that we exist in a simulation rests on the ability to objectify (apply an algorithm to) the infinite. To objectify requires one to be apart from that which is being objectified.

    But to be apart from the infinite means being outside of it, and "it" extends everywhere. Max Planck explained this simply with his "we can't get behind consciousness".

    In other words, the infinite is consciousness and it is "immathematical", it's the realm in which poets, writers, artists, creatives, inventors dwell (but not too long, lest they be locked up in a mental health care facility, as more often happens to artists and the like). It is a realm that cannot be thought (reasoned) it can only be felt.

    It is not a rational (computational) dimension. Both "reason" and "rational" have the Latin root "ratio" - in other words, the rational and the reasonable are subsets of a greater immathematical ground.

    I think Bohm came to this realization, when he wrote that "The actual operation of intelligence is thus beyond the possibility of being determined or conditioned by factors that can be included in any knowable law ... Intelligence is thus not deducible or explainable on the basis of any branch of knowledge (e. g. physics or biology). Its origin is deeper and more inward than any knowable order that could describe it."

    Artists get it. Bohm got it, as did Planck. Maybe those who conjecture the universe is a simulation will eventually "get it" as well, when they discover the poetic, the artistic within themselves.

    The writers of "Contact" also "got it" when they had Jodie Foster, in a state of reverie, lament "they should have sent a poet".

    ReplyDelete
  98. I cant believe I missed the news that epistemology had finally been sewn up.


    Let the kids have some fun.

    ReplyDelete
  99. Sabine, if it comes down to 'Why don't you read what I explained' you're either tired from the day and don't have enough energy for charity (understandable, happens to us all), or you just decided to read uncharitably.

    How do you prove the thing you're sitting in front of now is a computer? Then I'll know what you accept as a proof.

    ReplyDelete
  100. _Shorty,

    It is evident from your comment that you don't have the faintest idea what I'm talking about to begin with. The logic of your argument goes like this: Because I can calculate the outcome of a Bell-type experiment on my laptop this means quantum mechanics is a classical, local theory. Hey, I have broken physics! Please line me up for the Nobel!

    Look, I don't have to know how a computer works to tell you what you can't and can't do with classical, local operations. For me a computer is just another model for reality, and models for reality is what I deal with for a living. I am sincerely sorry to hurt your computer scientist's pride, but in the end it's all physics. Now go and look up Bell's theorem. If you manage to disprove it, please publish your results and then come back.

    I have no clue what you think my age has to do with any of that.

    ReplyDelete
  101. I said nothing about your age.

    I'll say this again. This:

    "If you try to build the universe from classical bits, you won’t get quantum effects, so forget about this – it doesn’t work."

    shows that *you* don't know what you're talking about. The fact that you don't know why doesn't mean it isn't so. Sorry, but that's the truth. You don't know what you're talking about, or that sentence wouldn't have come from you.

    ReplyDelete
  102. Sabine,

    As far as I get it, I think your main argument is the complexity of simulating the physical laws as we observe them. But is it not possible that the actual physical laws are much more complex, and we have no way to probe them (because we are in a computer) and what is our universe is actually a much smaller part of the world of the future beings? That is, they don't have to simulate the whole universe or all of its laws for us to be in the simulation.

    ReplyDelete
  103. I think of the simulation hypothesis as philosophical speculation and don't try to apply science to it.But if you want to do that I wouldn't disagree with your main points.

    You wandered off the main point into some other areas about artificial brains and consciousness.

    We will likely create (and to a big extent already have) hardware/software that do very specialized complex tasks - for example, drive a car. However, even when I drive a car, most of what I do is unconscious. Recently my consciousness is much more focused on listening to Warren Zevon while I commute to work and the actions of driving are almost automatic. Much of what the human brain does is not conscious.

    So what would it mean to simulate consciousness? Do we mean that the artificial brain would possess qualia? Or are we simply saying the artificial brain has acquired some critical threshold of abilities to perform complex tasks that it now seems to be conscious?

    ReplyDelete
  104. _Shorty,

    You're right to the extent that my sentence failed to spell out explicitly that I was assuming the underlying interaction is local. You can do it classically if non-local. In that case, however, nobody knows how to get quantum field theory - see my note added. If you believe that you can prove it can be done, please enlighten us how, because we poor and stupid physicists haven't figured out how to do it. It's pretty clear though from your previous comments that you don't even understand the problem.

    ReplyDelete
  105. Anything you understand can be modeled on a computer. Whether a computer/network fast enough and large enough exists today isn't relevant. And just because you don't understand it doesn't mean it is impossible for anyone to ever understand it. I know full well that you know more about physics than I ever will. This does not change the fact that anything that is understood can be translated to a computer algorithm and thus simulated on a computer. And, no, I'm not saying *we* understand everything that would be required to build a simulation of our universe. What I am saying is that if it were understood well enough it could be simulated. The only things we couldn't simulate would be the things we do not know enough about. "Classical bits", as you want to call them, are just fine and dandy. Modelling the behaviour you want is not limited by "classical bits." It doesn't matter if "classical bits" are not "quantum bits." You can still descrive quantum behaviour via "classical bits." You just need to know how to write a suitable algorithm. Just because you don't understand how to write it doesn't mean it is impossible to write. Anything can be modeled. Anything. As long as you understand what it is you're trying to model.

    ReplyDelete
  106. _Shorty,

    I give up. I merely post your last comment to demonstrate the communication failure.

    ReplyDelete
  107. Keep it simple then. The two-slit experiment's results are due to quantum effects, correct? As I understand what you're saying, you believe you cannot model that experiment on a computer with "classical bits." Is that not what you're saying? Because that sure seems to be what you're saying.

    ReplyDelete
  108. _Shorty,

    Seriously? After all I've explained you didn't even bother to look up Bell's theorem?

    ReplyDelete
  109. No need to look up anything. You think because you can't do something, nobody can. Not even someone with more knowledge and technology than you. *shrug* I honestly do not know how you cannot get that through your head. You are saying "I personally don't know how to do "A" right now. So nobody anywhere ever will be able to, either. Because the knowledge I possess right now says to me that it is impossible, and that means it will always be impossible." Scienticians. Heh.

    ReplyDelete
  110. _Shorty,

    As I said, you're welcome to try. But unless you manage to come up with a way to circumvent Bell's theorem, I can't take your claims seriously. (Neither will any other physicist.)

    ReplyDelete
  111. As I said, you just don't get it. Your argument is no different than someone from 15,000 years ago thinking it is impossible for people to fly, so nobody will ever fly. And yet, planes.

    The fact that you personally can't build a universe simulator right now because you can't see how it could possibly be done doesn't mean the universe you're in can't be a simulated one. It could be. You don't know. There's tons of crap you don't know. Just like everyone else. Your argument that it is impossible because you don't understand how it could be possible makes no sense. I'm no physicist, but I get that. Da plane, boss! Da plane!

    ReplyDelete
  112. "Because the knowledge I possess right now says to me that it is impossible, and that means it will always be impossible."

    a) That is a gross mischaracterization of what Dr. Hossenfelder has said.

    b) In fact, certain things *have* been proved to be impossible, and will always be impossible. For a trivial example, if N is an integer, N*N+1 (N-squared plus one) will never have three as a factor. (Therefore you should read and understand Bell's Theorem.)

    c) If your counter-position is that anything that seems impossible now might be possible in the future, that is an example of an unfalsifiable hypothesis (as long as the future of the human species still exists), which science has long known are not worth considering.

    ReplyDelete
  113. Interesting analysis.

    The interesting thing I find is that in order to valid the world as a "real object," we are using data from our measures. So how you use that data reinforces the belief that what is real, conforms to the data from the measures you use. But you create measure in order to validate? You see the circle?

    So data "as information" may be called bits? You may not like to call bits data, so is there some "other way" information can be represented?

    Consciousness, implies a range of perception that the mind is capable of? Any physical object, is focused reality conforming toward objectification. Consciousness creating measure is a subjective perception is what the mind created?

    ReplyDelete
  114. _Shorty,

    I am not saying it's not possible that we live in a simulation. I am saying that it's a non-trivial statement that isn't easy to make compatible with all we know about the laws of nature. Or some of us know, anyway.

    It is really remarkable that you continue to talk down to me after you've just publicly demonstrated your utter ignorance of even basic physics.

    ReplyDelete
  115. Hello Ms Hossenfelder,

    I read your interesting blog post. I have to say, I am interested in physics but being a mechanical engineer I can't claim to understand even a tiny percentage of what you are. I'm here to make a question on the topic. I remember this from when it got published a couple of years ago: https://arxiv.org/pdf/1210.1847.pdf
    It is supposed to be an effort to devise a scientific way that will lead to experimentation that will help us understand whether we live in a simulated universe or not. Not sure about the results, etc. Can't even understand the essence of the paper. Can you tell me if it holds any worth? Also, would there be any way that we can work our way scientifically to set up an experiment that will give us indications towards one way or the other? Because from your ramblings, I understand that you think we are not working on that direction, but follow a sallow approach of belief. Is it possible that we can prove one or the other, or do you feel this will remain a philosophical question that scientists should leave aside for now?

    Thank you :)

    ReplyDelete
  116. _Shorty

    "Anything can be modeled. Anything"

    Rubbish. Actually, "Complete and utter rubbish".

    Tell you what, why don't you disprove not only Bell's Theorem, proving that local reality is unable to explain the world we experience, but also Gödel' Incompleteness, Heisenberg's Principle of Indeterminism, and Chaitin's Randomness Theorem, in which he states "some mathematical facts are true for no reason, they are true by accident, or at random. In other words, God not only plays dice in physics, but even in pure mathematics, in logic, in the world of pure reason. Sometimes mathematical truth is completely random and has no structure or pattern that we will ever be able to understand."

    If you think you can model randomness, not some computer generated randomness, but the genuine thing, go for it.

    When you're done doing that, go off and model the infinite. Report back when you're done. After which you'll need to again model, and thereafter, yet again, and again and again (ad infinitum).

    ReplyDelete
  117. I continue to talk down to you because you don't understand everything you think you do, as smart as you think you are. I know something you don't know. And it is eating you up inside. And you don't like that. Because you think I'm not anywhere near as smart as you. And that's why you continue to respond. Because you hate it. And can't leave it be.

    I'll remind you, you don't know what knowledge I possess. You only know what I allowed you to see. You know, with my free will. hahahaha

    ReplyDelete
  118. _Shorty,

    I am posting your comments because you highlight the problem my post was alluding to. People talking about physics without knowing a thing about it to begin with. Do you realize at all that you're making an idiot out of yourself by proclaiming you can disprove Bell's theorem while at the same time demonstrating you don't even know what the theorem says?

    ReplyDelete
  119. And yet, you don't understand something so simple. I thought you were supposed to be smarter than me? Confounded by an "idiot." Hilarious. With every interaction you confirm the stuff your "peers" publicly say about you. Quite entertaining. :)

    ReplyDelete
  120. I winced when I heard Neil deGrasse Tyson supporting this harebrained idea.

    My problem with the simulation hypothesis is that - even if there is some way to simulate a large space including quantum effects, or even if there is some way to deceive the brains in the simulation into thinking this is what is happening, it would be prohibitively expensive computationally.

    In our universe, a simulator is necessarily much larger than that which is simulated. Wouldn't a computer builder in the outer universe have the same problem?

    The usual way we deal with this problem is to make a simulated universe extremely simple compared to the real world, and certainly no more complex than necessary. Thus, for example, a first-person shooter doesn't simulate the inner workings of a person; the model of a person is no more complex than a single (classical) molecule in term of its physical behavior, and things like air, walls and mountains are merely drawn, not physically simulated.

    Even if our universe were classical, it is hard to overstate how simple and inadequate our own computer simulations are compared to what we would need to accurately simulate, say, the inner workings of a tiny speck of dust. We can't simulate a single bacterium at the atomic level; simulating individual proteins is a big challenge. So our technology - which is perhaps far from theoretically optimum, but not *that* far - requires, what? A processor trillions of times larger than the space being simulated? Just for a classical simulation. Presumably the outer universe could have more a favorable ratio between the size of the computer and the amount of matter in the simulated space, but that ratio would surely remain above 1, wouldn't it?

    There are two possibilities. Either the "programmer" is trying hard to deceive scientists, or else the physical laws and facts inferred by scientists are real. And the facts are these: scientists have inferred a universe whose complexity at the smallest scales is dramatically, even unfathomably, greater than what is actually necessary for micro-organisms (let alone humans) to exist, and whose size at the largest scales is not only beyond our ability to imagine, but indistinguishable from infinite.

    It seems therefore that we can rule out the second possibility; surely the size of "the matrix" is not virtually infinite, especially since that implies it took a virtually infinite length of time to build it. The only remaining possibility is that the "programmer" (we should really call him "The Architect" - have you not seen The Matrix?) is intentionally deceiving us.

    But if one of the goals is to deceive scientists into thinking we are not in a simulation, why work so hard at it? If "the architect" were doing enough simplifications to make the simulation tractable (such as running a classical simulation with an "ether", no relativity, etc.) why bother to fool us into believing we're in a dramatically more complex universe than we are actually in? If our laws of physics were much simpler, all the processes of life could still be possible, and the simplifications wouldn't make it any harder to deceive us. The only things that really need to be "covered up" are simplifications that can only be explained by positing a computer that simulates our universe.

    And finally, what would be the point of deceiving us in the first place?

    Some people bizarrely seem to think that quantum effects are evidence of a lossy "compression algorithm" that makes the simulation more tractable. The reality of course is that quantum effects make everything more complex and harder to simulate, not easier. Evidence of lossy compression artifacts have never been seen.

    ReplyDelete
  121. This Shorty, huh? I could ask him why people are working so hard to build large and expensive quantum computers with modest, non-general-purpose computing abilities if classical computers are all one could ever need. I could point out that as a computer engineer who recently took a course in quantum computation, I might even have a clue about this. But I certainly have no desire to talk to him.

    However, if Bell's theorem just says "No physical theory of local hidden variables can ever reproduce all of the predictions of quantum mechanics" (Wikipedia) I don't see how it is necessarily a roadblock for making a simulation. The "outer" universe could play by different rules, allowing instant data transmission across arbitrary distance, for instance. What I can't fathom is why, if this is The Matrix, our universe would be unimaginably large and the laws of physics would be absurdly hard to simulate.

    ReplyDelete
  122. Qwertie,

    Yes, indeed! The 'real' universe could work by other laws than ours. And these more fundamental laws could be used to give rise to the laws we observe. Which is exactly what theoretical physicists have been working on for decades...

    ReplyDelete
  123. Hey Qwertie
    Just some questions here. Why would you say that our universe is unimaginably large? For whose imagination is this claim valid? For ours? Why would that matter for the creator of the simulation? This simulation may not even have to do with us at all. Did that occur to you at all? Also, why do you suggest that the simulation should have laws of physics that are easy to simulate? Do you suggest that what we see as "hard" would also be hard for the creator of such a simulation? I wouldn't think so personally. Also, you said something about "very expensive to simulate" on a previous comment. Yeah, having a smartphone in the 80ies would be absurdly costly, but now it costs just $50. You say you studied quantum computing. What if the creator uses something relevant to this technology, or even something more advanced? Wouldn't that make a simulation much much cheaper to run?

    ReplyDelete
  124. If there is such an entity as a creator who simulates 'all of it', then i would personally be more interested in the constitution of that creator, since he would be the only 'real' stuff around. Which means the simulation is the boring part of the quest, haha.

    ReplyDelete
  125. I agree with Sabine, I think the majority here are either ill-informed or ignorant of the deeper implication of quantum physics.

    E.g. _Shorty, and all those who "think" that we or someone may one day, or is simulating or modeling reality.

    As physicist Dr Bernard Haisch points out "the Leggett inequality that was recently measured ... rules out any possible interpretation other than consciousness creates reality when the measurement is made."

    Max Planck said simply, "we cannot get behind consciousness".

    Likewise David Bohm, "the operation of intelligence ... is deeper and more inward than any knowable order that could describe it"

    Likewise physicist Freeman Dyson "Quantum mechanics makes matter even in the smallest pieces into an active agent, and I think that is something very fundamental. Every particle in the universe is an active agent making choices between random processes"

    What the mechanically minded responding on this topic don't or refuse to appreciate is - as Dyson explains - all of reality is collapsing the wave-function, all the way down to the smallest particle, and as per David Deutsch's "shadow photons", those as well.

    No one will ever, ever, ever simulate all of the individual particles, including the shadow particles in the multi-verse collapsing the wave-function in their own special way, according to constraints and systems within which they operate.

    As Dyson also explains "consciousness is not just a passive epiphenomenon carried along by the chemical events in our brains, but is an active agent forcing the molecular complexes to make choices between one quantum state and another. In other words, mind is already inherent in every electron, and the processes of human consciousness differ only in degree but not in kind from the processes of choice between quantum states which we call 'chance' when they are made by electrons."

    I share Sabine's frustration, but due to the fact that those who are mechanically minded (those who believe or even conjecture that one day we or someone can simulate the process of choice being performed by each particle in the universe, and multiverse), and who have not availed themselves of the quantum evidence, are also people who are helping shape societies. And that affects all of us, to our detriment.

    For all those mechanically-minded folk, here's a question: What reaches into the infinite recesses of possibility to begin the collapse of the wave-function (from possibility to actuality). See my diagram on this https://stephenpirie.com/sites/all/files/simple-tools-fig.6b.png

    What "mechanism" can do that?

    What reaches into the unutterable, unspeakable, "immathematical" depths of existence, and does that, again and again, trillions of times each second?

    If you want an answer, the quickest pathway is to ask an artist, poet, or anyone who is intuitively, precognitively gifted. Then go and study the real implications of quantum physics, and marry the two (left and right hemispheres in your heads), rather than being imprisoned in your left-hemispherical world of numbers, names and notions.

    ReplyDelete
  126. "However, if Bell's theorem just says "No physical theory of local hidden variables can ever reproduce all of the predictions of quantum mechanics" (Wikipedia) I don't see how it is necessarily a roadblock for making a simulation." (Qwertie)

    I think the point was that the simulation would require qubits, not classical bits.

    As for computers getting better and cheaper, Moore's Law or any exponential law works in the earlier stages of growth and development, but cannot go on forever, and I think physics dictates that Moore's Law is just about done. (Circuit sizes in the range of small numbers of atoms.) Then going to qubits would require maintaining temperatures near absolute zero, which will add huge costs (outer space itself in Earth's vicinity is too hot for good efficiency). Finally, as mentioned in previous comments, the ability to simulate every parameter (mass, energy, spin, etc.) of every particle in the observable universe (so that wherever we look we find no glitches) is mathematically impossible in this universe. One must postulate a higher universe with magical capabilities, total unobservable and forever unknown to us, with incomprehensible motives (since if we want to simulate, say, the collision of two black holes for scientific purposes, there is no need to model ants as part of the simulation - or people either, which would also look like ants to such simulators). As also mentioned before, speculations involving unobservable, unknowable, and incomprehensible hypotheses are not encouraged in science - but knock yourself out.

    ReplyDelete
  127. The whole discussion strikes me as pretty absurd. The best known methods for studying the time evolution of quantum many body systems without serious approximations can do maybe 100 atoms at most. This only works by treating nuclei as point particles. If you need the dynamics of neutrons and protons you can at most do a small nucleus, say carbon. And if you want the dynamics of quarks and gluons there are no known methods at all.

    Of course we all hope for the advent of quantum computing. But you still need a physical system to encode a qbit. Indeed, the easiest way to simulate a quantum system is to make it, and let the quantum hamiltonian compute its own evolution. If our simulation overlords wanted to simulate a universe, they would first have to create a physical universe to run the simulation. What, exactly, would be the point of that?

    ReplyDelete
  128. JimV

    "the ability to simulate every parameter (mass, energy, spin, etc.) of every particle in the observable universe"

    Well, that assumes a certain interpretation of quantum theory that many (I believe a majority) of physicists would dismiss.

    As David Deutsch explains (in The Fabric of Reality) "Single-particle interference phenomena unequivocally rule out the possibility that the tangible universe around us is all that exists" - aka, we exist in a multiverse, not simply just that which is directly observable (noting that recent experiments have established the superposition of molecules containing around 5,000 atoms, if I recall correctly).

    Now, knowing each particle is in a superposition prior to observation, who's going to simulate those superpositions?

    Which also means per Thomas Schaefer above, that not only would "they would first have to create a physical universe to run the simulation" they would also have to create the multiverse, containing all those superpositions, for every photon, quark, lepton, electron, etc.

    As JimV adroitly suggested, "knock yourself out".

    ReplyDelete
  129. Furthermore, let's for the moment imagine that some higher-order entities were to simulate the multiverse, with all those endless superpositions.

    At Cambridge University, mathematician Noah Linden and physicist Sandu Popescu found that
    "in the typical quantum state occupied by any group of particles the links between the particles are mostly of a nonlocal character. Quantum theory isn’t just a tiny bit nonlocal. It’s overwhelmingly nonlocal. Nonlocality is the rule for our Universe"

    So even those doing the simulation are part of the nonlocal interconnectedness. In other words, they'd be part of the simulation, for everything effects everything.

    That "everything effects everything" I believe led physicist John Wheeler to suggest that we're also effecting the past -- in his delayed choice experiments, he envisaged (didn't get to complete it, I understand) using galactic lenses to confirm our choices now choose which past we experience all the way back to the Big-Bang. He concluded that we live in a participatory universe, wherein everything is helping/participating in the unfoldment/evolution/creation of an unimaginably vast self-organising system (hence why galaxies are known to distribute themselves fractally).

    Good luck simulating yourself, all the present multiverse, and the past ones as well (necessary for the foundational delayed-choice experimental results)

    ReplyDelete